LSAT and Law School Admissions Forum

Get expert LSAT preparation and law school admissions advice from PowerScore Test Preparation.

User avatar
 Dave Killoran
PowerScore Staff
  • PowerScore Staff
  • Posts: 5853
  • Joined: Mar 25, 2011
|
#49156
Setup and Rule Diagram Explanation

This is a Grouping: Defined-Fixed, Unbalanced: Overloaded, Numerical Distribution game.

Here is the initial scenario for the game:

PT24-Dec 1997 LGE-G4srd1.png

Because there are three different types of treatments, subscripts should be used to help track each treatment group.

The first rule establishes that all three antibiotics cannot be prescribed:

PT24-Dec 1997 LGE-G4srd2.png

The second rule reserves exactly one of the five treatments for a dietary regimen:

PT24-Dec 1997 LGE-G4srd3.png

These first two rules indicate that a Numerical Distribution is present. In fact, there are only two possible numerical distributions of the three treatment groups to the five prescriptions in this game:

PT24-Dec 1997 LGE-G4srd4.png

The third, fourth, fifth, and sixth rules are each conditional in nature:

PT24-Dec 1997 LGE-G4srd5.png

Note that rule 3 presents the contrapositive in order to eliminate the negative terms.

There are a number of powerful inferences that can be drawn from the rules:

Second and Third Rules Combined
The second rule indicates that exactly one dietary regimen is prescribed. The third rule indicates that if F is prescribed, then O—a dietary regimen—must be prescribed. Hence, if F is prescribed, no other dietary regimen besides O can be prescribed, and thus F cannot be prescribed with M or N.
Second and Last Rules Combined
The last rule indicates that if V is prescribed, then both H and M are prescribed. As M is a dietary regimen, if V is prescribed then no other dietary regimen besides M can be prescribed, and thus V cannot be prescribed with N or O.
Second, Third, and Last Rules Combined
From the second rule, only one dietary regimen can be prescribed. The third rule indicates that if F is prescribed, then O—a dietary regimen— is prescribed. The last rule states that if V is prescribed, then H and M—a dietary regimen—is prescribed. Thus, F and V cannot be prescribed together as they both require different dietary regimens.
By applying the five negative grouping inferences explained above, the following answer choices can be eliminated:

Question #18: Answers (A), (B), and (C)
Question #19: Answers (A), (B), and (C)
Question #20: Answers (B) and (D)
Question #21: Answers (A), (B), (C), and (D)
Question #22: Answers (A), (C), and (E)
Many of the remaining answers can be eliminated by a simple application of the rules.

This is the final setup for the game:

PT24-Dec 1997 LGE-G4srd6.png

In fact, due to the many rules and restrictions, there are only five solutions to this game:

PT24-Dec 1997 LGE-G4srd7.png

A student who identified each of these five possibilities could easily destroy the game. However, because the game can so easily be solved by using the inferences, and identifying the possibilities is somewhat time-consuming, we do not feel it is necessary to identify the possibilities.
You do not have the required permissions to view the files attached to this post.
 srcline@noctrl.edu
  • Posts: 243
  • Joined: Oct 16, 2015
|
#23242
Hello,

So here is my setup and rule diagram for this game? Is this correct?

----- ----
in out

1. 2 antibodies prescribed :arrow: remaining antibiotic cant be prescribed
(+)If remaining antibiotic is prescribed :arrow: the 2 antibodies cant be prescribed

O not prescribed :arrow: F cant be prescribed
(+) If F is prescribed :arrow: O is prescribed

Inference : F :dbl: O

If W is prescribed :arrow: F cant be prescribed
(+) F is prescribed :arrow: W is not prescribed

N and U are prescribed :arrow: G cant be prescribed
(+) If G is prescribed :arrow: N or U is not prescribed
Inference: G :dblline: N
G :dblline: U

If V prescribed :arrow: H and M are prescribed
(+)If H or M is not prescribed :arrow: V is not prescribed

I was only able to answer questions 18 and 20

Thankyou

Sarah
 Clay Cooper
PowerScore Staff
  • PowerScore Staff
  • Posts: 241
  • Joined: Jul 03, 2015
|
#23281
Hi Sarah,

Thanks for your question.

Your setup looks good - this is a simple grouping game, with an in-group and an out-group, as you have correctly identified and labeled. All good so far.

Your rules are accurate as well - you have correctly diagrammed each conditional rule, as well as its contrapositive. Good work.

Where you get off track a bit is with your inferences - neither of them is correct.

For the first inference, there is no double arrow; in other words, your inference seems to indicate that if O is prescribed, F must be; that is not the case. We have no rule that tells us that.

The second inference is also incorrect. It is true that, if G is prescribed, either N or U (or both) must not be prescribed; but it could be the case that G is prescribed, and N is prescribed, but U is not prescribed. That would violate your inference without violating any of the rules. Thus, the inference is not an accurate reflection of the rules and is incorrect.

When looking for inferences in this game, I would encourage you to consider the numerical limits on how many treatments from each category of treatment can or must or cannot be prescribed. That is where the inferences are to be found in this game. Beware converting causal arrows to double arrows erroneously; it is an easy mistake to make, I can tell you from experience, but it will totally derail your approach to the game.

Keep working hard!
 LSAT2018
  • Posts: 242
  • Joined: Jan 10, 2018
|
#46663
I diagrammed the conditional rules as the following:
Not O → Not F
F → O

W → Not F
F → Not W

N and U → G
Not G → Not N or Not U

V → H and M
Not H or Not M → Not V


For the conjunction of the fifth and sixth rules, it can be taken that there are three possibilities for each, right?
Not G → Not N or Not U (or Not N and Not U)

Not H or Not M → Not V
Not H or Not M (or Not H and Not M) → Not V


As for inferences, I focused on the implications of the second rule, 'There must be exactly one dietary regimen prescribed.'
For example, if O was selected, M and N would be out, meaning that V would be out.
And if M was selected, O and N would be out, meaning that F would be out.
And if N was selected, M and O would be out, meaning that V and F would be out, and the remaining are G, H, U and W, which would be impossible given the fifth rule. Hence, M or O must always be selected.

Are there any other inferences that I needed?
 ava17
  • Posts: 18
  • Joined: Jan 12, 2019
|
#61796
If we take the contrapositive of the fifth rule, does that mean: G :arrow: notN or notU?

Are there any inferences from this?
 Adam Tyson
PowerScore Staff
  • PowerScore Staff
  • Posts: 5153
  • Joined: Apr 14, 2011
|
#61835
LSAT2018 - one error in your diagram is in the rule about N, G, and U. Rather than:

N and U :arrow: G
it should be
N and U :arrow: G

That's because it says that IF N and U are prescribed, G cannot be (I switched the order and added emphasis, but it means the same thing)

That means that if G is prescribed, either N or U must not be.

Fix that rule and revisit your inferences, which mostly look good. For example, V and N will never be together, nor will V and O ever be together. Good inference about N - it can never be prescribed - but it's unclear how you got there given your error in the 5th rule. The way you have it written, a solution of NGHUW looks acceptable, but it should not be.

I happen to like doing this game as a template game, based on which dietary regimen is selected. Give that a try, too, and see if you make any additional inferences. There is another good one lurking out there for you to find!

ava17, you got that rule just right! If G is selected, then N or U must be out, maybe both. There is at least one inference related to that rule, but it comes from a combination of several rules working together. Nothing comes from it immediately, independent of the other rules. But, if you want to dig some, try this:

G and N in, U out

G and U in, N out

G in and N and U both out

I wouldn't do that much on a timed test, but it can be a good exercise in untimed practice. See what you get as a result! Have fun with it!

Get the most out of your LSAT Prep Plus subscription.

Analyze and track your performance with our Testing and Analytics Package.